Hey guys, this one is just for funnsies. So when dealing with an alternating series test, 3 requirements must be met, :
Alternating
u(sub n) ≥ u(sub n+1) for all n ≥ N, for some integer N
u(sub n) → 0 as n → ∞.
So I have been coming up with examples where of these are true, and one isnt. A...
Hi,
Is it possible to determine the force (energy) behind a 300dia blank when under a pressure of 0.5bar and subsequently how far the force (energy) may move the blank which has a mass of 16kg?
I know the volume and pressure of the air so can the stored internal energy be approximated by...
Hi friends ,
I am an electrical engineering student , interested in applying M.S in physics program in Europe . So i have started preparing for this , when i searched the web how to study for PGRE , most effective and common suggestion is go through the recommended books for each subject and...
First, because the series is positive term, we don't have to worry about absolute values. Now $\displaystyle \begin{align*} a_n = \frac{2n + 3}{4n^3 + n} \end{align*}$ and
$\displaystyle \begin{align*} a_{n + 1} &= \frac{2\left( n + 1 \right) + 3}{4 \left( n + 1 \right) ^3 + n + 1} \\ &=...
How would you answer the following question?
If ##(f\circ g)(x)=x##, then ##g## is the inverse function of ##f##. True/False?
This is on a test that I gave and I now think it is a bad question, but I'm tired and I want to hear some other people's impressions. The wording of the question is...
Bee Hossenfelder called attention to an interesting paper in this post:
A newly proposed table-top experiment might be able to demonstrate that gravity is quantized
http://backreaction.blogspot.com/2015/10/a-newly-proposed-table-top-experiment.html
Here's the paper...
Hi, I'm looking for the result of displacement controlled tensile test.
I want to know about the x-axis component and y-axis component of the result, not the whole data.
I read about some articles about these and they said displacement control is most common.
I've searched a lot but all I found...
I'm thinking of giving gre test
i don't know if i need only to give the subject test or general is must.
i'm thinking of getting into physics Ph.D program so Physics GRE test.
Please someone explain how am i supposed to give the test to be considered for a phd
thanks
According to wikipedia AQFT needs test functions so that the fields are distributions smeared on these functions. I'd want to know what are these test functions.
I read in Haag's book that they are fast decreasing functions defined on space time. They belong to the set S of Schwartz functions...
Homework Statement
Not surprisingly there is an old forum post on this topic:
https://www.physicsforums.com/threads/proving-force.46215/
However it doesn't contain the answer.
Reposted:
Two physics students are having a debate about the best way to define a force scale using rubber bands. Each...
1. Object 1 starts at 25 m and moves with a velocity of –5.5 m/s . Object 2 starts at 13 m and moves directly toward object 1. The two objects collide 0.65 s after starting. I need to find when they both collide and the velocity of Object 2
2. I don't know much but I know the displacement...
When interpreting the results of the two channel Bell test, I believe there were roughly 40 nanoseconds between when the particles were created and when they hit the polarized lens. It is believed that sometime during this time information passes between the entangled photons so that they match...
Given a sequence, how to check if it converges?
Assume the sequence is monotonic but the formula that created the sequence is unknown.
My first thought was if:
seq(n+2) - seq(n+1) < seq(n+1) - seq(n) , is always true as n->infinity then it is convergent.
Or in other words, if the difference...
Let's say I have some data and I want to test the hypothesis H_0 (only background) vs the hypothesis H_1 (bkg +signal).
I did that using the ##p##-value and I got with a Z-score and two different approaches (taking all the data or the data within some mass window) the results:
\it{p}_1 =0.105...
In slip test, we provide a constant 3 phase voltage across the stator of the machine. Then why does that voltage vary? Its like a variable inductor circuit due to different air gap length but basically if the voltage impressed is kept constant, why does the voltmeter show variation?
Is it like...
I have a question inspired by a recent thread that I did not want to hijack (https://www.physicsforums.com/threads/distributions-on-non-test-functions.831144/)
I realize that weaker requirements on the space of test functions results in a more restricted set of distributions. For example, if...
Homework Statement
I have three equations:
## F = ρw(x_0 y^2_0 - x_1 y^2_1) + \frac{1}{2} γ w (x^2_0 - x^2_1)## ----- 1
##y_0 = y_1 \frac{x_1}{x_0}## ----- 2
##\frac{y^2_0}{2} + gx_0 = \frac{y^2_1}{2} + gx_1## ------ 3
Homework Equations
N/A
The Attempt at a Solution
My goal is to have...
Hi guys,
The problem isn't with the question itself its more to do with the fact about how they got Z and R. Just need some clarification.
In every other AC power question. I have always worked out impedance with whatever parameters I had be it P/I^2 or V/I. I'm confused as to what the...
While this is the first Bell test that simultaneously addresses both the detection and the locality loophole, am I mistaken that this would still not be considered a loophole-free test?
Experimental loophole-free violation of a Bell inequality using entangled electron spins separated by 1.3 km...
Homework Statement
Alexander sled down a hill and on a snow-covered horizontal leveled ground. He wanted to calculate the highest velocity that the sled can reach. The hill is 3 meters high and 6 meters long. When Alexander sled down the hill he traveled 12,5 meters on the snow-covered ground...
I'm trying to make a check list of courses I need to study and review for the GRE Physics Subject Test. I have a few questions following the list. Thank-you for any help.
Engineering Physics I
Engineering Physics II
Modern Physics
Electricity & Magnetism
Statistical Mechanics & Thermodynamics...
Not sure that I've phrased the question correctly. If you have a series of p values from a series of tests, and they're all meant to be uniformly distributed, why do you have to do a KS test on that, and not another Chi-squared test?
The following is an extract from a test program's output:-...
Hi all
We are currently conducting Charpy V Notch Test to the Longitudinal Weld Seam of a Steel Pipe. The specimens we have tests are of two different size : full size (10mm x 10mm x 55mmL) and 2/3 size (6.7mm x 10mm x55mmL).
We have conducted the tests severally time at -30degC. The results...
I just came across the user instructions for my smoke detectors and am surprised. I've attached a small section of the instructions. They're from a mainstream manufacturer and labelled with a British Kitemark. You can see that I'm not supposed to test it with flame, smoke nor heat, and the...
Hi all,
I was reading a paper and I wasn't sure how they performed their statistic analysis. What do they mean by a chi-square test against the weighted mean? Is it a chi-square test between the actual data points and the weighted mean? If so, why do they us the function P(X^2,v)? I...
Hey you guys! I'm a med student from Sweden in need of some help!
I'm doing a summer course type-a thing in a lab, and I have to present some data from a project I've been doing here. Now, I've tested several genes' expression and I've compared those of an experimental group to a control group...
The problem:
Researchers conduct a study to test the effectiveness of a drug preventing a disease. Of 20 patients in the study, 10 are randomly assigned to receive the drug and 10 to receive a placebo. After 1 year, suppose 5 patients in the control group contract the disease, while 2 patients...
Given that we know the affected root and dermatome, can the nerve conduction test be used to localize the site of the nerve problem? I mean can it tell the distance of the pathology compressing on the nerve from the skin surface?
I applied at an internship for the position of a Junior Java Developer in Test and I am wondering if you can provide me some screening question examples?
The internship requires that I have strong knowledge in SQL and Java. I asked the recruiter to elaborate on the technical interview questions...
I found the interval of convergence for a hypergeometric series as |x| < 1, now I believe that I need to apply 'Gauss's test' to check the end point(s). For $ \left| x \right|=1 $ my $ \left| \frac{{a}_{n}}{{a}_{n+1}} \right| = \left|...
Hi,
I am currently working with a histogram (with 25 bins) that looks Gaussian and am trying to fit a function to it and compute its goodness of fit. The function I am using to fit to the histogram is a Gaussian (it looks like a good fit from visual inspection) and I am treating this as my...
Did they manage to do a loopholes free Bell test ? The best I got from google was an article from february that says no , they only did one where 2 out of 3 loopholes were eliminated in one test.
I will try to explain this with an analogy.
Let's have this equation:
x^2 =9
And let's assume I don't know algebraic methods to solve it, so I create a list using excel with different values. And I see that if I put x=4 it doesn't work, if I put x=5 it is even worse and so on. But If I put...
According to this link: http://tutorial.math.lamar.edu/Classes/CalcI/ShapeofGraphPtII.aspx
The second derivative test can only be applied if ##f''## is continuous in a region around ##c##.
But according to this link...
Homework Statement
[/B]
Here is an nth term test for determining divergence, I think I have it, but wanted another opinion --
1/34 + 1/35 + 1/36+ … + 1/1,000,034 -- IHomework Equations
∑(upper limit ∞)(lower limit n=0) 1/(n+34)
The Attempt at a Solution
1/34 + 1/35 + 1/36+ … + 1/1,000,034...
I have the sum,
$$\sum_{n=1}^{\infty} \frac{1}{n^{3}}\sin(n \pi x) \text{, where }0 \leq x \leq 1$$
I have to show that the series converges, so I'm going to use the Comparison Test.
$$ \text{If }0 \leq a_n \leq b_n \text{ then}$$$$\text{If }\sum b_n \text{ converges then }\sum a_n \text{ must...
Hi,
I am trying to use an integral test on the following series:
The sum from n=0 to infinity on 1/(n+1)^x where x>1
I know the process of using the integral test however i am unsure as to how to evaluate the integral with the x in the series :/
Thanks in advance!
Homework Statement
[/B]
You want to see if a coin is fair. You flip it 5 times and count the number of heads. If H is the number of heads obtained in five flips of the coin, what is the P-value of the test when H equals 4?
Homework Equations
None
The Attempt at a Solution
To solve this...
Let k be the number of real solutions to the equation e^x + x - 2 = 0 in the interval [0,1] and let n be the number of real solutions NOT on the interval [0,1]. Which of the following are true?
A) k = 0 and n = 1
B) k = 1 and n = 0
C) k = n = 1
D) k > 1
E) n > 1
Can anyone help me understand...
Below is a screen shot of a solution to a problem. The part I don't fathom is after the ratio test is applied to the denominator. How can, noting that an+1, (2n-1) become (2n-1)(2n+1) and not just (2(n+1)-1)=2n+1?
Thank you in advance
Homework Statement
Using the integral test determine whether or not the following sum diverges. $$ \sum_{n=1}^{\infty} \frac{e^n}{e^{2n}+9} $$
Homework Equations
The Attempt at a Solution
$$ \sum_{n=1}^{\infty} \frac{e^n}{e^{2n}+9}=\sum_{n=1}^{\infty} \frac{e^n}{({e^{n})}^2+9} \\
\int^{\infty}...
Hey,
I'm trying to look for a single test set-up for a dark matter-only simulation I and my friend are building. It's currently based on a particle-in-cell approach and we are calculating the Poisson equation and particle trajectories in a co-moving frame (so expansion of the Universe is taken...
The Legendre functions are the solutions to the Legendre differential equation. They are given as a power series by the recursive formula (link [1] given below):
##\begin{align}y(x)=\sum_{n=0}^\infty a_n x^n\end{align}##
##\begin{align}a_{n+2}=-\frac{(l+n+1)(l-n)}{(n+1)(n+2)}a_n\end{align}##...
Homework Statement
Determine all values of P for which the series ∑((-1)^(n-1))((ln(x))^p)/(5n) is convergent, expressing your answer in interval notation (Problem is shown in attached picture).
Homework Equations
Alternating Series Test: If {a_n} is positive and decreasing, and if the lim as...
I am a part C Mechanical Engineering student and have been undertaking a project investigating the strain sensitivity of a particular glass filled composite.
From my quasi-static results, the stress-strain curves seem to be reasonable and match that of the mechanical material properties...
Homework Statement
How can I calculate the ultimate stress? (I refere to the stress that it's in the green rectangle). I have a real graphic, I have two deformations, (it was 2 test, from the same material) and only one stress. The orange curve is Stress versus unit deformation 1, and the gray...
I am looking to determine fracture toughness for aluminium, from a punch test. I have the dimensions of the specimen and the load-displacement curve of the test. I have seen how to calculate Kic for brittle materials from a punch test but am struggling to find a formula for ductile materials...